1994 IMO Problems/Problem 1

Revision as of 23:38, 9 April 2021 by Math31415926535 (talk | contribs) (Created page with "Let <math>a_1, a_2, \dots a_m</math> satisfy the given conditions. We will prove that for all <math>j, 1 \le j \le m,</math> <cmath>a_j+a_{m-j+1} \ge n+1</cmath> WLOG, let <...")
(diff) ← Older revision | Latest revision (diff) | Newer revision → (diff)

Let $a_1, a_2, \dots a_m$ satisfy the given conditions. We will prove that for all $j, 1 \le j \le m,$

\[a_j+a_{m-j+1} \ge n+1\]

WLOG, let $a_1 < a_2 < \dots < a_m$. Assume that for some $j, 1 \le j \le m,$

\[a_j + a_{m-j+1} \le n\]

This implies, for each $i, 1 \le i \le j,$ \[a_i + a_{m-j+1} \le n\] because $a_i \le a_j$

For each of these values of i, we must have $a_i + a_{m-j+1} = a_{k_i}$ such that $a_{k_i}$ is a member of the sequence for each $i$. Because $a_i > 0, a_{k_i} > a_{m-j+1}$. Combining all of our conditions we have that each of $k_i$ must be distinct integers such that \[m-j+1 < k_i \le m\]

However, there are $j$ distinct $k_i$, but only $j-1$ integers satisfying the above inequality, so we have a contradiction. Our assumption that $a_j + a_{m-j+1} \le n$ was false, so $a_j + a_{m-j+1} \ge n+1$ for all $j$ such that $1 \le j \le m$ Summing these inequalities together for $1 \le j \le m$ gives \[2(a_1+a_2+ \dots a_m) \ge m(n+1)\] which rearranges to \[\frac{a_1+a_2+ \dots a_m}{m} \ge \frac{n+1}{2}\]